TestPrep - Medical

Ace your homework & exams now with Quizwiz!

A 72-year-old woman is found unresponsive in her poorly ventilated home. Her skin is flushed, hot, and dry, and her respirations are rapid and shallow. She is wearing a medical alert bracelet that states she is a diabetic and is allergic to sulfa drugs. What should you suspect? A) Heatstroke B) Anaphylactic shock C) Acute ischemic stroke D) Diabetic complications

A) Heatstroke The patient's signs and symptoms are consistent with classic heatstroke. Unlike exertional heatstroke, which usually affects otherwise healthy people who exert themselves in the heat for long periods of time, classic heatstroke commonly affects children and older adults, and it typically occurs when the patient is in a hot, poorly ventilated space for a prolonged period of time. Significant underlying medical problems (eg, hypertension, diabetes, heart disease) increase the patient's risk for classic heatstroke. Both exertional and classic heatstroke present with hot, flushed skin; however, patients with exertional heatstroke may have moist skin, usually from residual perspiration, whereas patients with classic heatstroke typically have dry skin.

A known woman with diabetes is found unresponsive. Her respirations are rapid and shallow; her skin is cool, clammy, and pale; and her pulse is rapid and weak. Which of the following would BEST explain the likely cause of her condition? A) Insulin overdose B) Excessive eating C) High blood sugar D) Failure to take insulin

A) Insulin overdose The patient has classic signs of insulin shock, a condition caused by a low blood glucose level (hypoglycemia). Common causes of insulin shock include insulin overdose (accidental or intentional), failure to eat (or not eating enough), and excessive exertion. The patient's symptoms are not consistent with diabetic ketoacidosis (DKA), a condition that results from a high blood glucose level (hyperglycemia). Signs of DKA include deep, rapid breathing (Kussmaul respirations) with a fruity or acetone breath odor, and warm, dry skin. Unlike insulin shock, which can result from an insulin overdose, DKA can occur if a patient fails to take his or her insulin or takes too little.

What is the correct dose, concentration, and route of epinephrine via EpiPen for anaphylactic shock in an adult? A) 0.3 mg; 1:1,000; intramuscular B) 0.15 mg; 1:1,000; intravascular C) 0.3 mg; 1:10,000; intramuscular D) 0.15 mg; 1:10,000; intravascular

A) 0.3 mg; 1:1,000; intramuscular In some EMS systems, EM Ts are trained to draw up and administer epinephrine from an ampule or a vial. In this case, the dose is 0.2 to 0.5 mg of a n ,000 solution (1 mg/mL) for adults and 0.01 mg/kg for children. Other systems carry epinephrine auto-injectors, or assist patients who have a prescription. The adult EpiPen delivers 0.3 mg of a J:1 ,000 solution, and the pediatric EpiPen delivers 0.15 mg of a J:2 ,000 solution. Whether drawn from an ampule or vial or delivered via EpiPen, the drug is administered via the intramuscular route. The J:1 ,000 concentration is very concentrated compared to J:10 ,000, and is therefore NOT administered by the intravascular route.

Which of the following signs is LEAST suggestive of a diabetic emergency? A) Bradycardia B) Tachycardia C) Combativeness D) Fruity breath odor

A) Bradycardia Bradycardia is not commonly associated with either hyperglycemia or hypoglycemia. Tachycardia and combativeness can occur in patients with hyperglycemia or hypoglycemia. A fruity or acetone breath odor, however, is a specific clinical finding in patients with hyperglycemic ketoacidosis (diabetic coma).

Shortly after being stung by a scorpion, a 20-year-old woman develops hives to her chest and arms. She is conscious and alert, but states that it is difficult to swallow. Her heart rate is 100 beats/min and her oxygen saturation is 94%. Which of her signs or symptoms should concern the EMT the MOST? A) Difficulty swallowing B) Hives to her chest and arms C) Heart rate of 100 beats/min D) Oxygen saturation of 94%

A) Difficulty swallowing The patient is experiencing an allergic reaction. Of the signs and symptoms listed, the fact that she is experiencing difficulty swallowing (dysphagia) should be the most concerning because it indicates swelling of the throat and upper airway; this is clearly the most life-threatening of her signs and symptoms. Urticaria (hives) are a clear indication of an allergic reaction; however, by themselves, they are harmless. The patient's oxygen saturation and heart rate are not grossly abnormal; however, you should closely monitor them.

A young woman reports significant weight loss over the last month, persistent fever, and night sweats. When you assess her, you note the presence of dark purple lesions covering her trunk and upper extremities. You should suspect: A) HIV infection. B) tuberculosis. C) rheumatic fever. D) end-stage cancer.

A) HIV infection. Weight loss, fever, and night sweats could indicate tuberculosis or HIV infection that has progressed to acquired immunodeficiency syndrome (AIDS); however, the dark purple lesions on the skin, which are called Kaposi's sarcoma, are malignant skin tumors and are a classic finding in patients in the later stages of AIDS.

You respond to a baseball field for a person who was struck by lightning. When you arrive, you see one patient who appears confused and is ambulatory; a second patient who is conscious, sitting on the ground, and holding his arm; and a third patient who is supine and motionless. After requesting additional responders, you should: A) assess the motionless victim and perform CPR and defibrillation if necessary. B) proceed to the ambulatory patient, sit him down, and assess his mental status. C) treat the conscious patients because the motionless patient is likely deceased. D) assess and begin treating the sitting patient first because he is obviously injured.

A) assess the motionless victim and perform CPR and defibrillation if necessary. The process of triaging multiple victims of a lightning strike is different than the conventional triage methods used during a mass-casualty incident. When a person is struck by lightning, respiratory or cardiac arrest, if it occurs, usually occurs immediately. Those who are conscious following a lightning strike are much less likely to develop delayed respiratory or cardiac arrest; they usually survive. Therefore, you should focus your efforts on those who are in respiratory or cardiac arrest. This process, called reverse triage, differs from conventional triage, where such patients would ordinarily be classified as deceased.

Which of the following patients with diabetes is the BEST candidate for oral glucose? A) confused patient who has cool, clammy skin B) A confused patient with suspected hyperglycemia C) A semiconscious patient with pale, clammy skin D) An unresponsive patient who took too much insulin

A) confused patient who has cool, clammy skin Oral glucose is indicated for patients with diabetes who are in insulin shock (hypoglycemic crisis) and for any patient with symptomatic hypoglycemia (as documented by glucometer). The patient must be conscious and alert enough to be able to swallow the glucose, which comes in a tube of gel. If the patient is unresponsive or otherwise unable to swallow the glucose, you should provide rapid transport, providing the appropriate airway management en route, and consider an ALS intercept.

After administering epinephrine to a woman in anaphylactic shock, her condition improves. The EMT should recognize that her clinical improvement is because epinephrine: A) constricts the blood vessels and dilates the bronchioles. B) decreases the heart rate, which improves cardiac output. C) blocks histamine receptors and stops the allergic reaction. D) causes fluid to shift back into the cells, which reduces edema.

A) constricts the blood vessels and dilates the bronchioles. The life-threatening complications of anaphylactic shock are caused by vasodilation, which results in hypo tension, and bronchoconstriction, which results in respiratory failure. Epinephrine reverses these effects because it stimulates alpha and beta receptors. Alpha receptor stimulation produces vasoconstriction, which increases the blood pressure. Beta receptor stimulation causes bronchodilation, which improves breathing. Epinephrine increases the heart rate, also through its beta stimulation properties. Epinephrine does not stop the allergic reaction; antihistamines (ie, diphenhydramine [Benadryl]), which bind to histamine receptors, actually terminate the reaction. Antihistamines also reduce the edema that occurs during a severe allergic reaction.

An 80-year-old woman has pain in the right upper quadrant of her abdomen and a yellow tinge to her skin. You should suspect dysfunction of the: A) liver. B) spleen. C) pancreas. D) gallbladder.

A) liver. Pain in the right upper quadrant and skin with a yellow tinge to it Uaundice) indicates a problem with the liver. Jaundice is the result of an excess level of bilirubin in the blood. Bilirubin, a yellow-colored substance, is the waste product that remains in the bloodstream after iron is removed from hemoglobin. One of the functions of the liver is to filter out waste, such as bilirubin, from the blood. If the liver is not functioning properly, bilirubin accumulates in the bloodstream. Dysfunction of the pancreas would result in possible fluctuations in the levels of blood glucose. Gallbladder inflammation (cholecystitis) typically produces right upper quadrant pain and/or referred pain to the right shoulder that occurs shortly after eating. Dysfunction of the spleen would cause left upper quadrant pain and/or referred pain to the left shoulder.

Common complaints in a patient with an infectious disease include: A) nausea, rash, fever, and shortness of breath. B) headache, low back pain, and arm numbness. C) chest discomfort, weakness, and vomiting. D) joint pain, muscle aches, and blurred vision.

A) nausea, rash, fever, and shortness of breath. Nausea is common with a number of infectious diseases, such as hepatitis. A rash should alert you to the possibility of sepsis, such as what could be caused by meningitis. Petechiae are tiny, circular, non-raised patches that appear on the skin or in a mucous membrane; they occur as the result of bleeding under the skin. A petechial rash may be present in a patient who is septic from a number of causes. Fever indicates inflammation or infection; in the context of other symptoms (ie, rash, nausea), you should be suspicious of an infectious disease. Shortness of breath can occur in patients with diseases such as tuberculosis or severe acute respiratory syndrome coronavirus 2 (SARS-CoV-2), the virus that causes coronavirus disease 2019 (COVID-19). Arm numbness, chest discomfort, and blurred vision are not common symptoms of an infectious disease.

A 23-year-old woman with a history of bulimia is vomiting large amounts of bright red blood and complains of pain in her upper abdominal area. What should you suspect? A) Mallory-Weiss tear B) Esophageal varices C) Gastroesophageal reflux disease D) Lower gastrointestinal bleeding

A) Mallory-Weiss tear Based on her history and symptoms, a Mallory-Weiss tear is the most likely underlying etiology. A Mallory-Weiss tear is a tear in the junction between the esophagus and the stomach, causing severe bleeding and potentially death. Primary risk factors include alcoholism and eating disorders. The patient has a history of bulimia, an eating disorder characterized by binging on large amounts of food, followed by self-induction of vomiting. Vomiting is the principle symptom of a Mallory-Weiss tear. In extreme cases, patients may experience upper abdominal pain and hemorrhagic shock. Esophageal varices also present with hematemesis; however, it is generally accompanied by difficulty swallowing and is more common in patients with alcoholism. Upper abdominal pain is uncommon with ruptured esophageal varices. Lower gastrointestinal bleeding would be expected to cause blood passage in the stool, not hematemesis. Gastroesophageal reflux disease (GERO) is condition in which gastric acids regurgitate from the stomach and into the esophagus. The principle symptom of GERO is heartburn.

Unresponsiveness, shallow breathing, and constricted pupils are indicative of which type of drug overdose? A) Narcotic B) Marijuana C) Barbiturate D) Amphetamine

A) Narcotic Signs of a narcotic (opiate/opioid) overdose from drugs such as heroin, morphine (Astromorph, Duramorph), meperidine (Demerol), and codeine include altered mental status; slow, shallow breathing; pupillary constriction (miosis), hypotension; and bradycardia. Narcotics are central nervous system depressants that, when taken in excess, suppress the vital functions necessary for life, such as breathing, heart rate, and blood pressure. Barbiturates produce the same effects; however, the pupils are typically dilated (mydriasis), not constricted. Marijuana and amphetamine drugs are central nervous system stimulants and, therefore, would cause the patient to become restless or even combative.

Which of the following statements regarding heatstroke is correct? A) Not all patients experiencing heatstroke have dry skin. B) Heatstroke is caused by a hyperactive sweating mechanism. C) Heatstroke is more likely to occur when the humidity is low. D) As core body temperature rises, the patient becomes more agitated.

A) Not all patients experiencing heatstroke have dry skin. Heatstroke occurs when the body is exposed to more heat than it can eliminate and normal mechanisms for eliminating heat, such as sweating, are overwhelmed. The core body temperature then rises rapidly to the point where tissues are destroyed. Heatstroke can develop in patients during prolonged vigorous physical activity or when they are in a closed, poorly ventilated, hot and humid space. High humidity impairs a person's ability to eliminate heat via the sweating mechanism. Many patients with heatstroke have hot, dry, flushed skin; however, early in the course of heatstroke, the skin may be moist due to residual perspiration, as with exertional heatstroke. As the core body temperature rises, the patient's level of consciousness decreases. Untreated heatstroke will result in death.

Which of the following signs and symptoms should the EMT expect to encounter in a patient with severe anemia? A) Pallor and shortness of breath B) Flushed skin and a slow pulse C) Jaundice and abdominal pain D) Hematemesis and nosebleed

A) Pallor and shortness of breath Anemia is a deficiency of red blood cells (erythrocytes). Recall that red blood cells carry hemoglobin, which binds to oxygen, in order to deliver oxygen to the body's cells and tissues. You would expect the severely anemic patient to be pale (because of fewer red blood cells). In addition, because the patient's oxygencarrying capacity is reduced, shortness of breath is also a common finding. In response to hypoxia, the patient would become tachycardic. Flushed (red) skin is not expected in anemic patients; you would expect flushing to occur if the patient had excessive red blood cell production (polycythemia). Jaundice (yellow skin) is observed in patients with liver disease or dysfunction, not anemia. Hematemesis and nosebleed may be observed in patients with low platelet levels (thrombocytopenia), not anemia.

A 45-year-old man reports shortness of breath that began suddenly. He tells the EMT that he has a condition called thrombophilia. What should you suspect? A) Pulmonary embolism B) Acute bronchospasm C) Congestive heart failure D) Intrapulmonary bleeding

A) Pulmonary embolism It is reasonable to believe that any one of the listed conditions could explain the patient's sudden shortness of breath. However, his history clearly leans toward a pulmonary embolism. Thrombophilia is an abnormality of blood coagulation that increases the risk of blood clots. Patients with this condition are often referred to as being hypercoagulable or prothrombotic. This case is a prime example of how knowing the patient's medical history can assist you in your differential diagnosis.

You are transporting a 30-year-old man who is experiencing an emotional crisis. The patient does not speak when you ask him questions. How should you respond to his unwillingness to speak? A) Remain sient until the pateint speaks to you. B) Continually encourage the patient to talk to you. C) Tell the patient that you cannot help if he will not talk. D) Do not speak to the patient, even if he begins to speak to you.

A) Remain sient until the pateint speaks to you. When dealing with a psychiatric patient who is silent and unwilling to speak to you, do not fear the silence. The patient simply does not wish to speak. You should not press the issue, because doing so may upset the patient. You should remain calm until the patient speaks to you, and then respond accordingly.

A 20-year-old man was stung on the arm by a bee and is experiencing local redness, pain, and swelling. You can see that the stinger is still in the man's arm. What should you do? A) Remove the stinger by gently scraping the skin with the edge of a rigid object B) Administer 0.3 mg epinephrine via EpiPen and then contact medical control C) Carefully grab the stinger with a pair of tweezers and gently lift it straight up D) Apply a pressure bandage to the site and elevate it above the level of the heart

A) Remove the stinger by gently scraping the skin with the edge of a rigid object If the stinger from an insect is still in place, attempt to remove the stinger by scraping the skin with the edge of a sharp, stiff object such as a credit card. Do not use tweezers or forceps to remove the stinger because this may squeeze more venom into the wound. After the stinger has been removed, gently clean the area with soap and water. Position the injection site slightly below the level of the heart, and apply an icepack to the area. Applying ice over the site may slow absorption of the venom, diminish swelling, and relieve pain. The patient in this scenario is experiencing a local reaction; therefore, epinephrine is not indicated at this time.

A 32-year-old man who was stung by a bee has diffuse hives, facial swelling, and difficulty breathing. When he breathes, you hear audible stridor. What does this indicate? A) Swelling of the upper airway structures B) Swelling of the lower airway structures C) Narrowing of the two mainstem bronchi D) Narrowing of the bronchioles in the lungs

A) Swelling of the upper airway structures This patient is experiencing a severe allergic reaction (anaphylaxis). Strid or, which is a high-pitched sound heard on inhalation, indicates swelling of the structures and tissues of the upper airway. If not promptly treated, the patient's airway may close completely, resulting in respiratory arrest. Narrowing of the bronchioles in the lungs causes wheezing, a whistling sound that may be heard during inhalation, exhalation, or both.

Which of the following is a complication associated with renal failure? A) Uremia B) Cystitis C) Renal calculi D) Low potassium

A) Uremia The kidneys play a major role in maintaining homeostasis, a stable internal environment. They preserve balance in the body by eliminating waste from the blood. When the kidneys fail, the patient loses the ability to excrete waste from the body, leading to a condition called uremia. This means that the waste product, urea, which is normally excreted into the urine, remains in the blood. Potassium is also eliminated in the urine; if the kidneys fail, the blood level of potassium case rise to life-threatening levels. Renal calculi (kidney stones) can block the passage or urine, causing injury to the kidneys; however, they are not caused by renal failure. Inflammation of the urinary bladder is called cystitis; while this can cause a urinary tract infection severe enough to injure the kidneys, it is not caused by renal failure.

Rapid transport of a patient who ingested a large dose of Tylenol is important because: A) an antidote may prevent liver failure if administered early enough. B) activated charcoal cannot be given to patients who ingested Tylenol. C) it takes only a small dose of Tylenol to cause cardiopulmonary arrest. D) liver failure usually occurs within 6 hours following a Tylenol overdose.

A) an antidote may prevent liver failure if administered early enough. Acetaminophen (APAP), the active ingredient in Tylenol, is a safe drug if taken as directed. However, ingestion of more than 140 mg/kg in an adult can cause liver failure and death. Symptoms of APAP overdose do not present acutely; it can take up to a week before signs of liver failure are apparent. The antidote for acetaminophen poisoning is acetylcysteine (Acetadote, Mucomyst), which is given only at the hospital. However, it must be given promptly if liver failure is to be avoided. Activated charcoal can be given to patients with APAP overdose; however, it is effective only if the drug is still in the stomach. After 1 to 2 hours following ingestion, activated charcoal would likely be ineffective.

A man is experiencing a severe allergic reaction after being stung by a scorpion. He does not have his own epinephrine; however, his wife is allergic to bees and has a prescribed epinephrine auto-injector. You should: A) provide rapid transport and consider an ALS intercept. B) assist the patient with the wife's prescribed epinephrine. C) administer an oral antihistamine and transport without delay. D) request an ALS unit to respond to the scene to administer epinephrine.

A) provide rapid transport and consider an ALS intercept. If a patient does not have a prescribed epinephrine auto-injector and is experiencing a severe allergic reaction, you should administer supplemental oxygen, assist the patient's ventilations if needed, and transport without delay. Closely monitor the patient's airway and breathing status en route and coordinate an ALS intercept if possible. If you carry an epinephrine auto-injector on your ambulance and your protocols allow you to administer it, do so without delay. Otherwise, the ALS unit will be able to administer epinephrine via the intramuscular or intravenous route. Never assist a patient with a medication that is not prescribed to him or her specifically. Antihistamines are given after epinephrine; in most systems, EMTs are not authorized to dispense oral medications other than glucose and aspirin.

You are dispatched to a residence for a 20-year-old man with respiratory distress. When you arrive, you find that the patient has a tracheostomy tube and is ventilator dependent. His mother tells you that he was doing fine, but then suddenly began experiencing breathing difficulty. You should: A) remove him from the mechanical ventilator and ventilate him manually. B) check the settings on the ventilator to ensure that it is functioning properly. C) detach the ventilator, suction the tracheostomy tube, and reassess the patient. D) remove the ventilator tubing and place an oxygen mask over the tracheostomy tube.

A) remove him from the mechanical ventilator and ventilate him manually. If a ventilator-dependent patient experiences a sudden onset of respiratory distress, you should first remove him or her from the mechanical ventilator and begin manual ventilation with a bag-mask device; attach the bag directly to the tracheostomy tube. If the patient improves, you will know that the problem was a malfunction with the mechanical ventilator. If the patient does not improve, the tracheostomy tube is likely plugged with thick mucus secretions and requires suctioning. Unless you are familiar with the mechanical ventilator (most EMTs are not), do not attempt to troubleshoot the device by checking the settings; this simply wastes time and could harm the patient if you do not know what you are doing.

You are transporting a 35-year-old man who has a history of alcoholism. He stopped drinking 4 days ago and is now disoriented, diaphoretic, and tachycardic. This patient is at risk for: A) seizures. B) hypothermia. C) hyperglycemia. D) schizophrenia.

A) seizures. A patient in alcohol withdrawal may experience frightening hallucinations or delirium tremens (DTs), a syndrome characterized by restlessness, fever, disorientation, sweating, and agitation. In severe cases, generalized seizures can occur; these seizures can be prolonged and can result in permanent brain injury or death. About 1 to 7 days after a person stops drinking or when alcohol consumption levels are suddenly decreased, DTs may develop. Patients with DTs are often dehydrated due to sweating, fluid loss, insufficient fluid intake, or vomiting, and their hallucinations can be extremely frightening. The alcoholic patient would more likely be hypoglycemic. Schizophrenia is a psychiatric illness; it is not caused by alcohol use or withdrawal.

A 50-year-old woman who is conscious and alert complains of a severe migraine headache. When caring for her, you should generally avoid: A) shining a light into her pupils. B) transporting her in a supine position. C) dimming the lights in the ambulance. D) applying ice packs to her forehead.

A) shining a light into her pupils. Patients with migraine or cluster headaches typically have photophobia (light sensitivity). Any type of bright light, especially if shone directly into the eyes, will cause the patient with a headache unnecessary severe pain. Dimming the lights in the ambulance and making the patient as comfortable as possible are the treatments of choice for a patient with a headache. Some patients benefit from ice packs applied to the forehead; just be sure to wrap the ice pack with roller gauze. Oxygen also should be administered as needed. Typically, the patient will prefer to lie supine or on the side.

A behavioral crisis is MOST accurately defined as: A) an acute psychiatric emergency characterized by violent behavior, mood swings, and a loss of connection to reality. B) a chronic mental health problem in which the patient experiences frequent thoughts of suicide or other self-destructive behavior. C) a persistent feeling of sadness, despair, or hopelessness that incapacitates the patient and prevents him or her from interacting socially. D) any reaction to an event that interferes with the activities of daily living or has become unacceptable to the patient, family, or community.

D) any reaction to an event that interferes with the activities of daily living or has become unacceptable to the patient, family, or community. The definition of a behavioral crisis or emergency is any reaction to an event that interferes with the activities of daily living (eg, bathing, dressing, eating) or has become unacceptable to the patient, his or her family, or the general community. Some patients react to an event with violent behavior or suicidal thoughts; others react with depression. Regardless of the reaction, it significantly interferes with the patient's life or is unacceptable to his or her family and/or the community.

A 40-year-old man with a history of depression and schizophrenia appears frightened and tells you that he sees snakes everywhere. You should: A) tell him that you do not see any snakes, but they are obviously scaring him. B) let him know that going to the hospital will keep him safe from the snakes. C) advise him that you suspect he has not been compliant with his medications. D) let him know that it is important for him to be transported as soon as possible.

A) tell him that you do not see any snakes, but they are obviously scaring him. When caring for any patient with a psychiatric crisis, your primary responsibility is to yourself, to stay safe. The patient in this scenario, although scared, is not violent. In cases such as this, you should be prepared to spend extra time with the patient; it may take longer to assess, listen to, and prepare the patient for transport. Just because he is experiencing an acute crisis does not mean that he has not been taking his medications; to make that assumption is merely speculation. You must be honest, reassuring, and nonjudgmental. Let the patient tell you how he or she is feeling in his or her own words, and acknowledge any auditory or visual hallucinations. Do not, however, play along with the hallucinations; this is cruel and simply reinforces that what the patient is hearing or seeing is real.

In general, you should avoid rewarming a frostbitten body part in the field if: A) the affected part could refreeze after rewarming. B) a paramedic is not present to administer analgesia. C) you are unable to obtain water that is at least 120°F. D) arrival at the emergency department will be delayed.

A) the affected part could refreeze after rewarming. Frostbitten body parts should not be rewarmed if there is a chance that they could refreeze after you have rewarmed them. If an extremity thaws and then refreezes, the amount of tissue and cellular damage may be worse than the damage caused by the initial freezing. A delay in getting the patient to the emergency department warrants rewarming. If rewarmed, the extremity should be immersed in water that is 105°F to 112°F. Analgesia would certainly be a comfort to the patient, although its absence does not negate rewarming a frostbitten body part in the field.

A 19-year-old man is experiencing hallucinations and paranoia after abusing an unknown substance. His heart rate is 170 beats/min, he complains of a headache, and he is experiencing muscle twitching. Which of the fo llowing substances would MOST likely explain his presentation? A) Heroin B) Bath salts C) Ketamine D)Valium

B) Bath salts Of the drugs listed, the patient's clinical presentation is the most consistent with use of synthetic cathinones (bath salts). Bath salts, not to be confused with products such as Epsom salt, are a class of drugs similar to MDMA (ecstasy). Cloud Nine and Ivory Wave are common streets names for bath salts. Bath salts produce euphoria, increased mental clarity, and sexual arousal. Most users snort the drug. Adverse effects include teeth grinding, loss of appetite, muscle twitching, lip -smacking, confusion, paranoia, headache, tachycardia, and hallucinations. Heroin (an opioid) and Valium (a benzodiazepine) are both central nervous system depressant drugs; therefore, they would not explain the patient's presentation. Ketamine (Ketalar) is a sedative-hypnotic drug; it would not explain the patient's signs and symptoms.

A 60-year-old man presents with an acute onset of confusion and slurred speech. Which of the following assessments would be the MOST likely to reveal the possible cause of his altered mental status? A) Oxygen saturation B) Blood glucose level C) Palpation of the skull D) Systolic blood pressure

B) Blood glucose level Altered mental status can be caused by a multitude of conditions, and they are too numerous to mention here. In this patient, the additional sign of slurred speech may help narrow your differential diagnosis; stroke and hypoglycemia both often present with slurred speech. Therefore, the patient's blood glucose level would most likely provide information as to the possible cause of his presentation; it should be assessed as soon as possible. A thorough assessment should be performed on this patient, including his oxygen saturation and vital signs. If the patient fell or struck his head, you should palpate his skull as part of your assessment.

Which of the following clinical presentations is the MOST consistent with an opioid overdose? A) Irregular pulse; rapid breathing; and diaphoresis B) Bradycardia; shallow breathing; and hypotension C) Bradycardia; hypertension; and pupillary dilation D) Tachycardia; hot, flushed skin; and severe agitation

B) Bradycardia; shallow breathing; and hypotension Opioids are in a class of drugs called narcotics. While they can be used as an effective means of analgesia, they can cause significant central nervous system (CNS) depression if taken in excess. Opioid toxicity depresses the respiratory drive, resulting in slow, shallow breathing. Other CNS depressant effects of toxicity include a decreased level of consciousness (including coma), bradycardia, hypotension, and constricted pupils (miosis). By contrast, CNS stimulant drugs, such as methamphetamine and cocaine, would cause hypertension, tachycardia, hyperthermia, and agitation or combativeness.

EMTs are assessing a man who is under the influence of an unknown substance. The patient is agitated, his BP is 180/100 mm Hg, his pulse rate is 150 beats/min, and his skin is hot to the touch. Which of the following drugs would MOST likely cause these signs and symptoms? A) Heroin B) Cocaine C) Alcohol D)Valium

B) Cocaine Cocaine is classified as a sympathomimetic, which means that it mimics the effects of the sympathetic nervous system. It is a very powerful stimulant. When overstimulated, the sympathetic nervous system produces signs and symptoms such as hypertension, tachycardia, and agitation or combativeness. Because the patient's metabolic rate is so high, more heat energy is produced, resulting in a high body temperature. Heroin, diazepam {Valium), and alcohol are all nervous system depressants. Patients who overdose on nervous system depressant drugs would be expected to present with a decreased level of consciousness (including coma), hypoventilation, bradycardia, and hypotension. The patient's clinical presentation is not consistent with a nervous system depressant overdose.

Which of the following drugs can cause or worsen excited delirium? A) Heroin B) Cocaine C) Ketamine D) Marijuana

B) Cocaine Excited delirium, also referred to as agitated delirium, is a condition caused by a severe increase in the metabolic rate, combined with impairment of cognitive function. The symptoms of excited delirium include hyperactive irrational behavior with possible vivid hallucinations, which can create the potential for violence. Common physical manifestations include hypertension, tachycardia, diaphoresis, hyperthermia, and dilated pupils. Drugs that increase metabolism, such as cocaine or methamphetamine (sympathomimetics), can cause or worsen excited delirium. The condition can lead to sudden death, usually from metabolic acidosis. Heroin, an opioid, depresses metabolism; therefore, it would not cause or worsen excited delirium. Ketamine, a sedativehypnotic, is commonly used to treat patients with excited delirium. Marijuana can cause changes in reality perception; however, the more potent sympathomimetics are far more common in inducing or worsening excited delirium.

Which of the following signs and symptoms would the EMT expect to encounter in a patient with organophosphate poisoning? A) Flushed skin, blurred vision, and tachycardia B) Excessive salivation, bradycardia, and vomiting C) Dilated pupils, jaundice, dry mouth, and nausea D) Hyperthermia, hypertension, and combativeness

B) Excessive salivation, bradycardia, and vomiting Organophosphates are found in industrial pesticides and insecticides; they are in a class of chemical called cholinergics. The parasympathetic nervous system is also referred to as the cholinergic nervous system, and it is responsible for slowing of the heart rate, as well as all your other resting functions (including the gastrointestinal tract). A patient with organophosphate poisoning would therefore be expected to present with bradycardia, hypersalivation, and other clinical signs of increased parasympathetic (cholinergic) activity. The clinical syndrome caused by organophosphate poisoning can be remembered with the mnemonic SLUDGEM, which stands for Salivation/Sweating; Lacrimation; Urination; Defecation, drooling, and diarrhea; Gastric upset and cramps; Emesis; and Muscle twitching and miosis (pinpoint pupils}. Another mnemonic is DUMBELS, which stands for Defecation, Urination, Miosis, Bradycardia, Emesis, Lacrimation, and Salivation.

Which of the following patient presentations would MOST likely require the EMT to don a high-efficiency particulate air (HEPA) respirator? A) Fever, weight loss, and jaundice B) Fever, a cough, and night sweats C) Jaundice, vomiting, and weakness D) Nuchal rigidity, fever, and headache

B) Fever, a cough, and night sweats Fever, a cough, and night sweats are signs of tuberculosis (TB). If they are observed in a patient, you should don a high-efficiency particulate air (HEPA) respirator (N95 or higher) because TB is transmitted via the droplet (aerosolization) route. The HEPA respirator will trap the bacterial particles and prevent you from inhaling them. Jaundice is a sign of liver disease or dysfunction, not TB. Nuchal rigidity, especially with fever, is a sign of meningitis; it is not a sign of TB. A HEPA N95 should also be worn when caring for a patient with suspected COVID-19.

Which of the fo llowing conditions would be the LEAST likely to be present in a patient who was submerged in water? A) Spinal injury B) Hyperglycemia C) Laryngospasm D) Gastric distention

B) Hyperglycemia Many factors can contribute to or result from a submersion injury (eg, drowning, near-drowning). It is not uncommon for a person to experience a spinal injury after diving head first into shallow water, especially if he or she is under the influence of alcohol. When a swimmer panics, he or she initially swallows large amounts of water, resulting in gastric distention. Gastric distention can cause aspiration if the patient regurgitates water during rescue breathing; protect the airway! During the panic phase, the victim expends a tremendous amount of energy (and glucose) by flailing around in the water, possibly resulting in hypoglycemia. Inhaling even a small amount of fresh or salt water can severely irritate the larynx, which sends the muscles of the larynx and vocal cords into spasm (laryngospasm), resulting in airway blockage and hypoxia.

Which of the following is a later sign of hepatitis? A) Fatigue B) Jaundice C) Loss of appetite D) Fever and vomiting

B) Jaundice Early signs and symptoms of viral hepatitis include loss of appetite (anorexia), vomiting, fever, fatigue, and muscle and joint pain. Jaundice (yellow sclera and skin) and right upper quadrant abdominal pain are not common early manifestations of hepatitis; they usually develop within 1 to 2 weeks into the disease process.

Hypoglycemia and acute ischemic stroke can present similarly because: A) the most common cause of stroke is hypoglycemia. B) both oxygen and glucose are needed for brain function. C) the majority of stroke patients have a history of diabetes. D) both are caused by low levels of glucose in the blood.

B) both oxygen and glucose are needed for brain function. Although stroke and hypoglycemia are two distinctly different conditions, their signs and symptoms are often similar. This is because the brain requires both oxygen and glucose to function normally. An acute ischemic stroke is caused by a lack of oxygen to a part of the brain due to a blocked cerebral artery, whereas hypoglycemia (low blood glucose level) deprives the entire brain of glucose. In either case, the patient presents with signs of impaired brain function (ie, slurred speech, weakness, altered mental status). Both conditions may lead to permanent brain damage or death if not treated promptly.

A patient who overdosed on methamphetamine would be expected to have all of the following clinical signs, EXCEPT: A) agitation. B) bradycardia. C) hypertension. D) dilated pupils.

B) bradycardia. Methamphetamine, an upper, stimulates the central nervous system, causing it to release excessive amounts of adrenaline (epinephrine). Epinephrine increases heart rate and blood pressure; therefore, the patient would experience tachycardia and hypertension. Other signs of methamphetamine overdose, which also indicate an adrenaline surge, include pupillary dilation, agitation, and hyperthermia. Bradycardia would be expected in patients who have overdosed on drugs that suppress-not stimulate-the central nervous system (eg, narcotics, benzodiazepines, barbiturates).

Immediately following a generalized motor seizure, most patients are: A) apneic. B) confused. C) hyperactive. D) awake and alert.

B) confused. After a generalized motor seizure, the patient typically will be confused, sleepy, or, in some cases, combative. This is referred to as the postictal phase. The patient's level of consciousness typically improves within 30 minutes. In many cases, the patient's respirations will be fast (tachypnea) following a seizure; this is the body's attempt to eliminate excess carbon dioxide that accumulated in the blood during the seizure. Tachycardia is also common immediately following a generalized motor seizure.

Active rewarming of a patient with moderate or severe hypothermia should be avoided in the field because: A) the risk of inadvertently inducing hyperthermia is too high. B) rewarming too quickly can cause a fatal cardiac dysrhythmia. C) it is painful for the patient and you cannot give analgesic drugs. D) active rewarming has been shown to cause severe hypertension.

B) rewarming too quickly can cause a fatal cardiac dysrhythmia. When caring for a patient with hypothermia, your goal is to prevent further heat loss; this involves removing wet clothing, applying warm blankets, and allowing the patient's body temperature to rise gradually and naturally (passive rewarming). If the patient is moderately or severely hypothermic, you should not try to rewarm him or her actively (placing heat on or into the body). Rewarming too quickly may cause a fatal cardiac dysrhythmia, such as ventricular fibrillation (VF). Active rewarming may also cause rewarming shock, a condition in which the blood vessels dilate when heat is applied to the body, resulting in significant hypotension. For these reasons, active rewarming should be performed only in the controlled setting of a hospital.

A young man experienced a syncopal episode after working in the heat for several hours. He is conscious and alert; has cool, clammy skin; and complains of nausea and lightheadedness. You should: A) provide rapid cooling. B) transport him on his side. C) give him cold water to drink. D) advise him to go home and rest.

B) transport him on his side. The patient is experiencing heat exhaustion and should be transported to the hospital for evaluation, especially since he experienced a syncopal episode (fainting). Because he is nauseated, he should not be given anything to drink and should be placed on his side during transport to prevent aspiration if he vomits. Loosen any restrictive clothing that may trap heat. Rapid cooling is indicated for patients with heatstroke, the signs of which include an altered mental status and hot, flushed skin (dry or moist).

A 39-year-old man asks you to take him to the hospital because has had a fever, headache, and diarrhea for the past 2 days. His blood pressure is 120/60 mm Hg, his pulse is 110 beats/min, and his respirations are 16 breaths/min. You should: A) ask him if he has a history of HIV infection or hepatitis. B) transport him to the hospital in a position of comfort. C) request an ALS ambulance to the scene to start an IV line. D) advise him that he can drive himself to his family physician.

B) transport him to the hospital in a position of comfort. Although the patient is likely experiencing the flu, there are other diseases, some of which are communicable, that can cause similar symptoms. The patient is requesting EMS transport; failure to comply constitutes abandonment. Although he is tachycardic, the remainder of his vital signs are stable; therefore, requesting an ALS ambulance to the scene to start an IV is not necessary. Simply transport him in a position of comfort and monitor him en route. If the patient is infected with HIV or hepatitis, he may choose to voluntarily disclose that information. However, to inquire about infection with such diseases is unethical.

You are called to an assisted living center where an attendant found a 72-year-old man unresponsive. The patient had recent hip surgery and has been taking Vicodin for pain. His respirations are slow and shallow, and his pulse is slow and weak. You should: A) apply high-flow oxygen via a nonrebreathing mask. B) ventilate the patient with a bag-mask device. C) administer intranasal naloxone at 0.4 mg. D) apply the AED and analyze his cardiac rhythm.

B) ventilate the patient with a bag-mask device. Vicodin is a combination of hydrocodone and acetaminophen (APAP}, the active ingredient in Tylenol. Hydrocodone is a narcotic analgesic. When taken in excess, it can suppress the central nervous system and cause respiratory depression, bradycardia, and hypotension. Initial management of any patient who has overdosed on a medication of this type is to ensure a patent airway and support breathing. Because the patient is breathing inadequately (slow and shallow), immediate ventilation assistance is needed. If you carry naloxone (Narcan) and are authorized to administer it, do so. Naloxone binds to opiate receptor sites and reverses the effects of narcotic drugs. If you are not authorized to administer naloxone, consider requesting a paramedic intercept, especially if your transport time will be lengthy. The AED is not indicated for this patient; it is applied only to patients in cardiac arrest.

Which of the following is a physiologic effect of epinephrine when used to treat anaphylactic shock? A) As a vasodilator, it increases the blood pressure. B) As a vasoconstrictor, it lowers the blood pressure. C) As a bronchodilator, it improves the patient's breathing. D) As an antihistamine, it blocks chemicals that cause the reaction.

C) As a bronchodilator, it improves the patient's breathing. Epinephrine possesses dual effects. As a bronchodilator, it relaxes the smooth muscle of the bronchioles and improves the patient's breathing. As a vasoconstrictor, it constricts the blood vessels and increases the patient's blood pressure. Diphenhydramine (Benadryl) is an antihistamine; it blocks H1 histamine receptor sites, which blocks the release of the chemicals (histamines) that are causing the allergic reaction.

A 23-year-old woman with sickle cell disease reports a sudden onset of severe pain in her lower extremities. Which of the following is the MOST likely cause of her symptoms? A) Red blood cells are being destroyed at an abnormal rate. B) Spontaneous bleeding is occurring within the muscles. C) Blood flow is diminished because of vascular occlusion. D) Blood clots have formed because of excessive platelets.

C) Blood flow is diminished because of vascular occlusion. Sickle cell disease is an inherited blood disorder in which the red blood cells are misshapen and take on a sickle appearance. The sharp and misshapen red blood cells lead to dysfunction in oxygen binding and unintentional clot formation. These unintentional clots may lead to a condition called vasoocclusive crisis. The patient's symptoms indicate that this is what has happened in her lower extremities. The blood clots in her lower extremities did not form because of an excessive platelet count (thrombocythemia) because sickle cell disease is not a condition of excessive platelet production. Spontaneous bleeding is not the cause of her symptoms because sickle cell disease is not a condition of low platelets (thrombocytopenia). Sickle cell disease is not a condition of abnormal red blood cell destruction.

Which of the following mechanisms cause respiratory and circulatory collapse during anaphylactic shock? A) Bronchodilation and vasodilation B) Bronchodilation and vasoconstriction C) Bronchoconstriction and vasodilation D) Bronchoconstriction and vasoconstriction

C) Bronchoconstriction and vasodilation During anaphylaxis, histamines released from the immune system cause two negative effects that result in shock (hypoperfusion): vasodilation, which causes the blood pressure to fall , and bronchoconstriction, which impairs breathing.

Which of the fo llowing conditions would MOST likely cause flushed skin? A) Blood loss B) Hypothermia C) Exposure to heat D) Low blood pressure

C) Exposure to heat Whenever the body temperature rises (ie, heat exposure, fever) , the peripheral blood vessels dilate, which draws warm blood to the skin and gives it a flushed (red) appearance. Blood loss, shock, low blood pressure (hypotension), and hypothermia generally cause the skin to become pale; these conditions cause peripheral vasoconstriction, which shunts blood away from the skin.

Which of the fo llowing structures is responsible for regulating body temperature? A) Cerebrum B) Cerebellum C) Hypothalamus D) Medulla oblongata

C) Hypothalamus The hypothalamus, which is located within the brainstem, regulates body temperature by acting as the body's thermostat. During a heat-related emergency, the hypothalamus can "reset" the body's normal temperature to a much higher temperature in response to the environment and the body's inability to eliminate heat.

Which of the following statements regarding the function of insulin is correct? A) It stimulates the liver to release glucose into the bloodstream. B) It promotes the entry of glucose from the cells into the bloodstream. C) It facilitates the uptake of glucose from the bloodstream into the cells. D) It causes the pancreas to produce glucose based on the body's demand.

C) It facilitates the uptake of glucose from the bloodstream into the cells. Insulin is a hormone produced by the beta cells in the islets of Langerhans of the pancreas. It promotes the uptake of glucose from the bloodstream into the cells, where it is used in the production of energy. Glucagon, a hormone produced by the alpha cells in the pancreas, facilitates the conversion of glycogen to glucose (glycogenolysis) in the liver. The liver does not produce glucose; it produces glycogen, a complex sugar that the body cannot utilize until it has been converted to glucose, a simple sugar.

Which part of the assessment process would MOST likely provide clues as to the possible underlying cause of a medical patient's chief complaint? A) Vital signs B) Mental status C) Medical history D) Primary assessment

C) Medical history The primary assessment identifies immediate threats to the patient's life, thereby allowing you to intervene in order to prevent further patient deterioration. The patient's vital signs and mental status also provide clues as to the patient's overall hemodynamic status; however, there are multiple causes of abnormal vital signs and mental status changes. The patient's history would provide you with the most information regarding the possible underlying cause of the patient's chief complaint. For example, discovering that an unresponsive patient (many conditions can cause unresponsiveness) has a history of diabetes would increase your index of suspicion for hypo- or hyperglycemia. A patient with chest pain (many conditions can cause chest pain) in a patient with recent coronary stent placement would increase your index of suspicion for cardiac ischemia.

A 31-year-old man presents with uncontrolled twitching of his left arm that began shortly after he experienced an odd taste in his mouth. He denies loss of consciousness and states that this has happened to him before. What should you suspect? A) Absence seizure B) Generalized seizure C) Simple partial seizure D) Complex partial seizure

C) Simple partial seizure Seizures are classified as being generalized or partial. Generalized seizures are further classified as being tonic-clonic (formerly called grand mal) or absence (formerly called petit mal) seizures. Generalized tonic-clonic seizures result from abnormal electrical discharges from large areas of the brain, usually involving both hemispheres. They are characterized by unconsciousness and generalized severe twitching of the body's muscles; they last several minutes or longer. Absence seizures are characterized by a brief lapse of consciousness in which the patient seems to stare and does not respond to anyone. Absence seizures do not involve any changes in motor activity. Partial (focal) seizures begin in one part of the brain and are classified as simple and complex. In a simple partial seizure, no change occurs in the patient's level of consciousness. Patients may report numbness, weakness, or dizziness. The senses may also be involved; the patient may report visual changes and unusual smells or tastes. A simple partial seizure may also cause twitching of the extremity muscles that spreads slowly from one part of the body to another. In a complex partial seizure, the patient has an altered mental status and does not interact normally with his or her environment. This type of seizure results from abnormal discharges from the temporal lobe of the brain. Other signs may include lip smacking, eye blinking, and isolated convulsions or jerking of the body or one part of the body, such as the arm. The patient may experience unpleasant smells and visual hallucinations, exhibit uncontrollable fear, or exhibit repetitive physical behavior such as constant sitting or standing.

Which of the following would MOST likely cause a rapid drop in a patient's blood glucose level? A) Mild exertion after eating a meal B) Eating a meal after taking insulin C) Taking too much prescribed insulin D) Forgetting to take prescribed insulin

C) Taking too much prescribed insulin Compared to diabetic coma (hyperglycemic ketoacidosis, hyperglycemic crisis), insulin shock (hypoglycemia, hypoglycemic crisis) has a rapid onset. It is commonly caused when a patient accidentally takes too much prescribed insulin. Insulin is a fast -acting drug that rapidly causes glucose to exit the bloodstream and enter the cells. Other common causes of hypoglycemia include taking a regular dose of insulin but not eating or taking insulin and exercising heavily. Eating a meal after taking insulin typically does not cause a significant change in the patient's blood glucose level. If a person fails to take his or her insulin, glucose will not be able to enter the cells and will accumulate in the bloodstream (hyperglycemia).

Which of the following signs and symptoms are MOST characteristic of hyperglycemic ketoacidosis? A) Cool, clammy skin and a slow onset B) Cool, clammy skin and a rapid onset C) Warm, dry skin and a slow onset D) Warm, dry skin and a rapid onset

C) Warm, dry skin and a slow onset Hyperglycemic ketoacidosis (diabetic coma) is characterized by a dangerously high blood glucose level (hyperglycemia); slow onset; warm, dry skin (from dehydration); Kussmaul respirations, which are deep and rapid; and breath with a fruity or acetone odor. Insulin shock results from a low blood glucose level (hypoglycemia) and is characterized by a rapid onset; altered mental status; and cool, clammy skin.

A 48-year-old man became acutely hypoxic, experienced a seizure, and is now postictal. The MOST effective way to prevent another seizure is to: A) place him in the recovery position. B) give him oral glucose if he can swallow. C) administer high-flow supplemental oxygen. D) dim the lights in the back of the ambulance.

C) administer high-flow supplemental oxygen. You should administer high-flow oxygen to all patients who are actively seizing and to patients who experienced a seizure and are postictal. This is especially true if the seizure was caused by hypoxia. Increasing the oxygen content of the blood, which minimizes hypoxia, may prevent another seizure. The recovery position is appropriate for uninjured patients with a decreased level of consciousness and adequate breathing; it will help maintain the airway and facilitate drainage of secretions from the mouth, but will not prevent another seizure. Oral glucose may prevent another seizure if hypoglycemia was the cause of the seizure. You should dim the lights in the back of the ambulance to help prevent any seizure, not just those that are caused by hypoxia.

Naloxone improves ventilation in a patient who overdosed on an opioid because it: A) neutralizes the drug in the bloodstream. B) allows more oxygen to bind to hemoglobin. C) binds to receptor sites and prevents drug uptake. D) dilates the bronchioles and opens the lower airways.

C) binds to receptor sites and prevents drug uptake. Naloxone (Narcan) is an opiate/opioid receptor antagonist. This means that it binds to opiate/opioid receptors in the body and prevents uptake of the drug by the central nervous system. As a result, the drug loses its influence over the respiratory centers and ventilation improves. Naloxone does not neutralize or break down the drug. Beta-2 agonists such as albuterol cause bronchodilation, thus opening the lower airways; naloxone has no effect on the bronchioles. Oxygen-hemoglobin binding increases when the patient's ventilations improve; naloxone does not directly do this.

A patient who overdosed on an opioid would be expected to present with: A) tachycardia. B) dilated pupils. C) bradypnea. D) hyperpnea.

C) bradypnea. As with all opioids, overdose causes depression of the central nervous system (CNS), resulting in a decreased level of consciousness, bradypnea (slow breathing), shallow depth of breathing (reduced tidal volume), bradycardia, and hypotension. Hyperpnea (deep breathing) would not be present in a patient who overdosed on an opioid. In an opioid overdose, the pupils are typically constricted (miosis). Barbiturates, such as phenobarbital, are also CNS depressants and cause the same symptoms seen with opioid overdose. However, the patient's pupils are typically dilated (mydriasis), not constricted.

When treating an unresponsive man who was struck by lightning, you should: A) begin CPR at once if he is apneic and pulseless. B) apply full spinal precautions before moving him. C) ensure that you and the patient are in a safe place. D) manually stabilize his head and open his airway.

C) ensure that you and the patient are in a safe place. Contrary to popular belief, lightning can (and does) strike in the same place twice. After lightning strikes, the ground remains electrically charged for a period of time; this increases the chance of a second strike within a short period of time. You must first ensure that you and the patient are safe by moving to a sheltered area, preferably indoors. After you have ensured the safety of yourself and the patient, begin treatment as dictated by the patient's condition. Do not let the life you save, or attempt to save, TAKE your own!

A 73-year-old man presents with confusion; cool, pale, clammy skin; absent radial pulses; and a blood pressure of70/40 mm Hg. The patient's wife tells you that he has had abdominal pain for a week and began vomiting a coffee-ground substance yesterday. His past medical history includes hypertension and gastric ulcer disease. Your MOST immediate concern should be that: A) his blood glucose level is probably too high. B) he is bleeding from his gastrointestinal tract C) he is in shock and requires prompt transport. D) his condition requires surgery within 2 hours.

C) he is in shock and requires prompt transport. The patient is likely bleeding from his gastrointestinal (GI} tract. Although this is a serious condition, it is not a condition you can treat; internal bleeding cannot be controlled in the field. You can, however, treat his signs and symptoms of shock by administering high-flow oxygen and keeping him warm by covering him with a blanket. Therefore, this should be your most immediate concern. Furthermore, the patient requires prompt transport to the hospital where he can receive definitive care, which may or may not involve surgery. It is highly unlikely that his blood glucose level is too high; he does not have a history of diabetes.

Patients who abuse opioids by injecting them are also at risk for: A) hepatitis A. B) tuberculosis. C) hepatitis C. D) schizophrenia.

C) hepatitis C. Patients who abuse opioids by injecting them often share needles with other people. This significantly increases their risk for contracting bloodborne pathogens such as hepatitis C and human immunodeficiency virus (HIV). Hepatitis A is spread by the fecal-oral route; it is not a bloodborne pathogen. Tuberculosis is spread by the droplet (aerosolization) route; it is not a bloodborne pathogen. Schizophrenia is a spectrum of psychiatric illnesses; there is no correlation between opioid abuse and schizophrenia.

The MOST obvious way to reduce heat loss from radiation and convection is to: A) move away from a cold object. B) wear a thick wind-proof jacket. C) move to a warmer environment. D) increase metabolism by shivering.

C) move to a warmer environment. In a cold environment, the body has two ways of staying warm: generating heat (thermogenesis) and reducing heat loss. Radiation is the transfer of heat by radiant energy. The body can lose heat by radiation, such as when a person stands in a cold room. Convection occurs when heat is transferred to circulating air, as when cool air moves across the body's surface. A person standing in windy cold weather, wearing lightweight clothing, is losing heat to the environment mostly by convection. The quickest and most obvious way to decrease heat loss from radiation and convection is to move out of the cold environment and seek shelter from wind. Shivering increases the body's metabolism and is a mechanism for generating heat, not reducing heat loss. Layers of clothing trap air and provide excellent insulation; thus, layered clothing decreases heat loss better than a single, thick jacket. Conduction is the direct transfer of heat from a part of the body to a colder object by direct contact, as when a warm hand touches cold metal or ice. The most obvious way to decrease heat loss by conduction is to remove your hand from the cold object.

When caring for a patient with severe hypothermia who is in cardiac arrest, you should: A) avoid using the AED. B) hyperventilate the patient. C) perform BLS and transport. D) perform rescue breathing only.

C) perform BLS and transport. Patients with severe hypothermia who are in cardiac arrest should be managed with basic life support (chest compressions and ventilations), passive external rewarming (ie, removal of wet clothing, applying warm blankets), and rapid transport to the hospital, where they can be actively rewarmed. Because cold muscle is a poor conductor of electricity, defibrillation, may be less effective until the patient's body temperature has been increased. However, you should not withhold defibrillation if it is indicated. Hyperventilation should be avoided, as this may increase intrathoracic pressure and impair blood flow back to the heart.

When restraining a violent patient, you should make sure that: A) at least two EMTs restrain the patient. B) the patient is restrained using maximal force. C) someone talks to the patient during the process. D) consent for restraint has been obtained from a family member.

C) someone talks to the patient during the process. If a violent patient needs to be restrained, you must ensure the presence of at least four people (one per extremity). One of the EM Ts should continuously talk to the patient to explain what is happening, even if the patient is not listening. Restraint is a last resort used to protect the EMT as well as the patient. Consent is not needed from a family member prior to restraining the patient. Just enough force to effectively restrain the patient is all that is required to prevent causing unnecessary injury.

You receive a call for a 54-year-old woman who is having a seizure. When you arrive at the scene, the patient is actively seizing. Her husband tells you that she has a history of seizures and that she has been seizing continuously for 20 minutes. It is MOST important for you to recognize that: A) her seizure could be the result of a low blood glucose level. B) the patient may have stopped taking her seizure medication. C) the patient is hypoxic and needs oxygen as soon as possible. D) her husband's description is consistent with status epilepticus.

C) the patient is hypoxic and needs oxygen as soon as possible. There are many causes of seizures, including noncompliance with prescribed anticonvulsant medications, hypoglycemia, stroke, head trauma, and poisoning, among others. Regardless of the underlying cause, it is most important to recognize that patients who are actively seizing, especially if the seizure is prolonged, are hypoxic and need supplemental oxygen as soon as possible; in many cases, it may be necessary to assist the patient's ventilations. The husband's description of the seizure (eg, continuous for 20 minutes) is consistent with status epilepticus, which is all the more reason to recognize that she is severely hypoxic. Seizure-related deaths are the result of cerebral hypoxia.

A 50-year-old woman reports a rash and itching to her trunk and arms that began a day after she started taking penicillin. Her breath sounds are clear to auscultation, she denies shortness of breath, her oxygen saturation is 96%, and her BP is 132/74 mm Hg. Treatment should include: A) high-flow oxygen and transport. B) epinephrine and rapid transport. C) transport only with close monitoring. D) warm compresses to relieve the itching.

C) transport only with close monitoring. The patient is obviously experiencing an allergic reaction, but she is not in anaphylactic shock. Her breath sounds are clear to auscultation, she is breathing without difficulty, her oxygen saturation is above 94%, and her blood pressure is stable. She requires little more than transport to the hospital with continuous monitoring en route. Epinephrine is not indicated, nor is supplemental oxygen. Warm compresses would make her itching worse, not better. Consider applying cool packs for itch relief.

The signs and symptoms of an allergic reaction to a bee sting are caused by: A) failure of the immune system to produce antibodies that render the bee's venom inactive. B) an inability of the immune system to recognize the bee's venom as being a foreign substance. C) the direct toxic effects of the bee's venom on the cardiovascular and respiratory systems. D) an exaggerated immune system reaction in which chemicals are released in response to the venom.

D) an exaggerated immune system reaction in which chemicals are released in response to the venom. An allergic reaction, an exaggerated immune response to any substance, is not caused directly by an outside stimulus, such as venom released during a bite or sting. Rather, it is a reaction that occurs when the immune system recognizes the stimulus as being foreign; in response, the immune system releases chemicals (histamines and leukotrienes) to combat the stimulus. An allergic reaction is not caused by a failure of the immune system to produce antibodies; rather, it occurs because the immune system produces them. A bee's venom does not have a direct toxic effect on the body; the chemicals released in response to the venom, however, can cause life-threatening conditions such as respiratory failure and shock.

When insulin levels in the blood remain high: A) a fruity odor can be detected on the patient's breath. B) the cells starve for glucose and begin to metabolize fat. C) the patient urinates excessively and becomes dehydrated. D) glucose is rapidly taken out of the blood to fuel the cells.

D) glucose is rapidly taken out of the blood to fuel the cells. Insulin is a hormone that promotes the cellular uptake of glucose from the bloodstream. If insulin levels remain high, such as when a diabetic inadvertently takes too much insulin, glucose is rapidly taken out of the blood to fuel the cells. This leads to low circulating blood glucose levels (hypoglycemia) and a condition called insulin shock. By contrast, if insulin levels are too low, such as when a diabetic forgets to take his or her insulin, glucose cannot enter the cells and pools in the bloodstream (hyperglycemia). In the absence of glucose, the cells begin to metabolize fat, which produces ketoacids (diabetic ketoacidosis [OKA]). The respiratory system attempts to eliminate ketoacids from the blood with an increased rate and depth of breathing (Kussmaul respirations) that is accompanied by a fruity or acetone breath odor. Hyperglycemia causes the patient to eliminate excess water from the body through urination (diuresis), resulting in dehydration.

A 29-year-old man with abdominal pain tells the EMT that he has hemophilia A. This means that: A) his red blood cells break down rapidly. B) he will lose blood at a much faster rate. C) his risk for a pulmonary embolism is high. D) he will bleed for a longer period of time.

D) he will bleed for a longer period of time. Hemophilia A is a rare disease caused by a deficiency of Factor VIII, which is an essential component of blood clotting. The body relies upon an intact clotting system in order to spontaneous stop bleeding (hemostasis). Hemophilia impairs the process of hemostasis because the patient's ability to create a clot following an injury is decreased. Hemophilia also increases the risk of spontaneous bleeding, including in the brain. Patients with hemophilia bleed longer, not faster. Patients whose blood clots too quickly (not slowly) are at risk for a pulmonary embolism. Hemophilia does not affect the red blood cells, or how quickly they break down (hemolysis).

Activated charcoal may be indicated following an ingestion of: A) lithium. B) gasoline. C) antifreeze. D) ibuprofen.

D) ibuprofen. Activated charcoal binds (adsorbs) to specific toxins and prevents their absorption by the body. The toxins are then carried out of the body in the stool. Activated charcoal is not indicated, nor will it be effective, for patients who have ingested alkali poisons, cyanide, ethanol (gasoline), iron, lithium, methanol (wood alcohol), mineral acids, organic solvents, and ethylene glycol (antifreeze). Patients who ingested large quantities of ibuprofen, acetaminophen, and aspirin would be typical candidates for activated charcoal, provided that they are conscious and alert and can protect their own airway.

You are at the scene where a man panicked while swimming in a small lake. Your initial attempt to rescue him should include: A) throwing a rope to the victim. B) rowing a small raft to the victim. C) swimming to the victim to rescue him. D) reaching for the victim with a long object.

D) reaching for the victim with a long object. General rules to fo llow when attempting to rescue a patient from the water include "reach, throw, row, and then go." In this case, you should attempt to reach the victim by having him grab hold of a long object, such as a stick or pole. If this is not possible or unsuccessful, throw the victim a rope or flotation device (if available). If these are not available, row to the patient in a small raft (if available). Going into the water to retrieve the victim is a last resort. The rescuer must be a strong swimmer because patients who are in danger of drowning are in a state of blind panic and will make every attempt to keep themselves afloat, even if it means forcing the rescuer underwater.

A patient with a core body temperature of 94°F would MOST likely present with: A) decreased muscle activity. B) an altered mental status. C) joint and muscle stiffness. D) shivering and pale skin.

D) shivering and pale skin. Mild hypothermia is defined as a core body temperature (CBT) between 93.2 degrees F and 96.8 degrees F (34 degrees C and 36 degrees C). Mildly hypothermic patients are usually alert and shivering in an attempt to generate heat through muscular activity. Their skin is typically pale due to the body's constriction of blood vessels at the skin to retain heat. In moderate hypothermia (CBT between 86 degrees F and 93.2 degrees F [30 degrees C and 34 degrees C]), shivering stops and muscular activity decreases. As the CBT falls further, all muscle activity stops. In severe hypothermia (CBT 86 degrees F [30 degrees C]), vital functions decrease (eg, level of consciousness, BP, pulse, respirations) and the patient is at risk for life-threatening cardiac dysrhythmias. The muscles become rigid and the patient appears stiff. The patient may appear dead; although a pulse is present, it may not be palpable.

A 16-year-old, 125-pound man ingested a bottle of aspirin approximately 20 minutes ago. Medical control orders you to administer activated charcoal in a dose of 1 g/kg. How much activated charcoal should you administer? A) 51 g B) 54 g C) 57 g D) 60 g

First, you must determine the patient's weight in kilograms (kg). Either of the following formulae can be used to convert pounds to kilograms: Formula 1: weight (in pounds)/ 2.2 = weight in kg. Formula 2: weight (in pounds)/ 2 minus 10% = weight in kg. On the basis of the above formulae, a 125-pound patient weighs 57 kg. Using formula 1, the equation is as follows: 125 (weight in pounds)/ 2.2 = 56.81 (57 [rounded to the nearest tenth]). Using formula 2, the equation is as follows: 125 (weight in pounds)/ 2 = 62.5 (63 [rounded to the nearest tenth] - 6.3 (10% of 63) = 56.7 (57 [rounded to the nearest tenth]). Since the drug order is for 1 g/kg, you should administer 57 g of activated charcoal to your 125-pound patient.

The two MOST important steps in treating a patient with a contact poisoning are: A) determining when the exposure occurred and irrigating the patient's entire body with copious amounts of water. B) donning a pair of gloves and performing a physical examination to determine if a significant exposure occurred. C) immediately washing any dry chemicals off of the patient's skin and removing his or her clothing as soon as possible. D) avoiding self-contamination and removing the irritating or corrosive substance from the patient as rapidly as possible.

D) avoiding self-contamination and removing the irritating or corrosive substance from the patient as rapidly as possible. When caring for a patient with a contact (skin surface) exposure to a poison, the two most important steps in management are avoiding contaminating yourself and removing the substance from the patient's skin as rapidly as possible. The level of personal protective equipment (PPE) you use depends on the type of chemical the patient was exposed to. After ensuring your own safety, remove all of the patient's clothing that has been contaminated, thoroughly brush off any dry chemicals, and then flush the skin with water. Always brush dry chemicals off the skin before irrigating with water; failing to do so may increase the amount of damage caused by the chemical. A physical exam of the patient should be performed only after he or she has been properly decontaminated.

In the patient with diabetes, insulin shock typically presents with: A) dry skin and a slow onset. B) dry skin and a rapid onset. C) clammy skin and a slow onset. D) clammy skin and a rapid onset.

D) clammy skin and a rapid onset. In the patient with diabetes, insulin shock (hypoglycemic crisis) presents with cool, clammy skin and a rapid onset. The brain is critically dependent on glucose and responds quickly when the body is in short supply. Diabetic coma (hyperglycemic crisis) typically presents with warm, dry skin and a slow onset, sometimes occurring over a period of days.

A 24-year-old woman presents with a rash to her left leg and swollen, painful knee joints. She tells you that she and her friends returned from a hiking trip in the mountains a week ago. She is conscious and alert with a blood pressure of 112/62 mm Hg, a pulse of 84 beats/min, and respirations of 14 breaths/min. What should you suspect? A) Tetanus B) Lyme disease C) An allergic reaction D) Rocky Mountain spotted fever

B) Lyme disease The patient's symptoms and her history of a recent hiking trip are consistent with Lyme disease, which is the result of a tick bite. Ticks can carry two infectious diseases: Lyme disease and Rocky Mountain spotted fever. Both are spread through the tick's saliva, which is injected into the skin when the tick attaches itself. The first symptom of Lyme disease, a rash that may spread to several parts of the body, begins about 3 days after the bite of an infected tick. The rash may eventually resemble a target or bull's-eye pattern in one-third of patients. After a few more days or weeks, painful swelling of the joints, particularly the knees, occurs. If recognized and treated promptly with antibiotics, many patients recover completely. Rocky Mountain spotted fever occurs within 7 to 10 days after being bitten by an infected tick. Its symptoms include nausea, vomiting, headache, weakness, paralysis, and possibly cardiopulmonary failure. Tetanus is not causing the patient's symptoms; patient's with tetanus experience severe muscle spasms (tetany). An allergic reaction would present within a few minutes to hours following exposure to an offending agent; this patient's symptoms are not consistent with an allergic reaction.

A 42-year-old man presents with fever, a severe headache, and a stiff neck. He is conscious, but confused. His wife tells you that he does not have any medical problems and does not take any medications. What should you suspect? A) Influenza B) Meningitis C) Tuberculosis D) Acute stroke

B) Meningitis Meningitis is an inflammation of the protective coverings of the brain and spinal cord (meninges). Common signs and symptoms of meningitis include fever, headache, neck stiffness (nuchal rigidity), and vomiting. An altered mental status is common in severe cases. Meningococcal meningitis, caused by a bacterium, is the most contagious and potentially fatal type of meningitis. The patient's signs and symptoms are not consistent with acute stroke, tuberculosis (TB}, or influenza (the flu). Although fever is common with both TB and the flu, neither causes neck stiffness. Acute stroke may be associated with a headache, especially a hemorrhagic stroke; however, stroke patients typically do not have a fever.

Law enforcement requests your assistance to evaluate a young woman whom they believe has overdosed. The patient is conscious, is displaying bizarre behavior, and is very restless. Her BP is 170/90 mm Hg and her heart rate is 130 beats/min. You see needle tracks on both of her arms. Which of the following would MOST likely explain her presentation? A) An opioid B) Methamphetamine C) A sedative-hypnotic D) A benzodiazepine

B) Methamphetamine The needle tracks on the patient's arms indicate opioid abuse, most likely heroin; however, her signs and symptoms are not consistent with those caused by an opioid or any other central nervous system (CNS) depressant (eg, benzodiazepine [a sedative-hypnotic]). Signs of CNS depression include a decreased level of consciousness, hypoventilation, bradycardia, and hypotension. Her signs and symptoms are consistent with a sympathomimetic (upper) overdose. Sympathomimetic drugs mimic the effects of the sympathetic (fight or flight) nervous system; therefore, restlessness or hyperactivity, hypertension, tachycardia, and pupillary dilation are common findings in patients who overdose. Amphetamine and methamphetamine (eg, ice, speed, crack) are examples of sympathomimetics.

Which of the following signs or symptoms would you MOST likely encounter in a patient with an inflamed gallbladder? A) Vomiting of dark red blood B) Pain in the right shoulder C) Bruising around the umbilicus D) Pain to the right lower quadrant

B) Pain in the right shoulder Inflammation of the gallbladder is called cholecystitis. Classic symptoms include right upper quadrant pain, usually shortly after a meal, and referred pain to the right shoulder. Nausea and vomiting may also occur. Hematemesis (vomiting blood) is a sign of gastrointestinal bleeding, not cholecystitis. Bruising around the umbilicus (Cullen sign) indicates blood in the peritoneal cavity. Pain in the lower right abdominal quadrant is common in patients with appendicitis, not cholecystitis.

A 50-year-old man reports severe pain to both upper abdominal quadrants. He states that the pain moves to his back and that he is nauseated and has been running a fever. His wife advises that he does not take care of himself and that he drinks a lot of alcohol. What should you suspect? A) Esophagitis B) Pancreatitis C) Cholecystitis D) Appendicitis

B) Pancreatitis Pancreatitis, inflammation of the pancreas, can be caused by an obstructing gallstone, alcohol abuse, and other diseases. Severe pain may present in the upper left and right abdominal quadrants and often radiates to the back. Other symptoms include nausea and vomiting, abdominal distention, and tenderness. Complications like sepsis or hemorrhage can occur, in which case your assessment may also reveal fever and/or tachycardia. Esophagi tis occurs when the esophageal lining becomes inflamed by infection or from the acids in the stomach. Symptoms of esophagi tis include a burning sensation in the chest (heartburn), pain with swallowing, and the feeling of an object stuck in the throat. The patient's symptoms are not consistent with esophagi tis. Cholecystitis, inflammation of the gallbladder, typically presents with right upper quadrant pain with referred pain to the right shoulder; the symptoms often appear shortly after a meal. The patient's symptoms are not consistent with cholecystitis. Patients with appendicitis can also present with nausea, vomiting, and fever; however, the pain is generally located around the umbilical area and radiates to the lower right abdominal quadrant. The patient's symptoms are not consistent with appendicitis.

A woman with type 1 diabetes presents with deep, rapid breathing; tachycardia, dehydration; and an altered mental status. Which of the following would MOST likely explain her clinical presentation? A) Her blood sugar level is less than 60 mg/dL. B) She has not taken her insulin in several days. C) She took her insulin but has not eaten today. D) She has an infection that lead to dehydration.

B) She has not taken her insulin in several days. The patient's clinical presentation is consistent with hyperglycemic ketoacidosis. Of the options listed, the only logical explanation is that she has not taken her insulin in several days. Without insulin, glucose cannot enter the cell to make energy, so it pools in the blood to excessive levels (hyperglycemia). Excess blood glucose levels cause the body to eliminate large amounts of water via the kidneys, resulting in dehydration. In the absence of glucose, the cells will metabolize fat, which produces ketoacids. The respiratory system will attempt to eliminate these ketoacids from the body by increasing the rate and depth of breathing (Kussmaul respirations). Had she taken her insulin but did not eat, her blood glucose level would be lower than normal. Although an underlying infection cannot be ruled out, the primary cause of her problem is failure to take her prescribed insulin.

Which of the following statements regarding lightning-related injuries is correct? A) The majority of patients who are struck by lightning die, even if CPR is provided immediately. B) The cardiovascular and nervous systems are most commonly injured during a lightning strike C) Because of the high electrical energy associated with lightning, full-thickness burns are a common finding. D) Multiple fractures, including those of the cervical spine, are the most common cause of lightning-related deaths.

B) The cardiovascular and nervous systems are most commonly injured during a lightning strike The cardiovascular and nervous systems are most commonly injured during a lightning strike; therefore, respiratory or cardiac arrest is the most common cause of lightning-related deaths, especially if CPR is delayed. A lightning strike typically induces asystole (cardiac standstill); however, this spontaneously resolves in some people or can be reversed with early, high-quality CPR. The energy associated with lightning is composed of direct current (DC) of up to 200,000 amperes and a potential of 100 million volts or more; however, the duration of a lightning strike is short. Therefore, skin burns are usually superficial; full-thickness (third-degree) burns are rare. Lightning causes massive contraction of all the body's muscles, potentially resulting in long bone and spinal fractures. Although this can clearly increase morbidity, it is not the most common cause of lightning-related deaths.

A 60-year-old man presents with a right-sided headache and weakness to his left side. He is conscious and alert. His blood pressure is 190/100 mm Hg, pulse is 88 beats/min, and respirations are 14 breaths/min. His past medical history includes diabetes, arthritis, and hypertension. Which of the following should concern you the MOST? A) His blood glucose level B) The presence of hemiparesis C) His current blood pressure D) The location of his headache

B) The presence of hemiparesis Headaches are a common medical complaint and have numerous causes, most of which are not life threatening. Common causes of a headache include migraine and cluster headaches, sinusitis, and muscle tension, among others. In some patients, however, a headache can signal a serious problem. In this patient, you should be most concerned with his unilateral weakness (hemiparesis); this is not a common finding in patients with otherwise benign headaches and suggests a serious underlying problem (eg, intracranial bleeding, brain tumor). The fact that his headache is to the right side of his head and his weakness is to the left side of his body suggests that the problem lies in the right side of the brain. It is doubtful that his blood glucose level (BGL) is abnormal; he is conscious and alert. However, it would not be unreasonable to assess his BGL. His blood pressure, which is significantly elevated, could be causing his headache or may be the result of other factors, such as noncompliance with his antihypertensive medication, anxiety, or pain.

A 28-year-old woman has severe lower quadrant abdominal pain. When assessing her abdomen, you should: A) ask her where the pain is located and palpate that area first. B) ask her where the pain is located and palpate that area last. C) auscultate for bowel sounds for approximately 2 to 5 minutes. D) encourage the patient to lie supine with her legs fully extended.

B) ask her where the pain is located and palpate that area last. Assessment of a patient's abdomen includes asking where the pain is located and then palpating that area last. Palpating the painful area first may interfere with the rest of your assessment because of the significant pain the patient will be in. Bowel sounds are of little value in the field and generally are not included in the abdominal assessment. Patients with severe abdominal pain typically prefer to lie on their side with their knees drawn up into their chest (fetal position). Moving them from this position will aggravate their pain.

A 30-year-old man with a history of diabetes is found unresponsive on his couch. His skin is cool and clammy, his pulse is rapid and weak, and his breathing is rapid and shallow. His blood glucose level reads 20 mg/dL. The EMTs should: A) place oral glucose in between his cheeks and gums. B) assist his ventilations and transport without delay. C) request a paramedic unit to administer his insulin. D) place him on his side, keep him cool, and transport.

B) assist his ventilations and transport without delay. A blood glucose level of 20 mg/dL is dangerously low. Combined with the patient's other signs and symptoms, this presentation is classic for insulin shock. However, because the patient is unresponsive, you cannot administer oral glucose because he could aspirate it. His rapid, shallow (ineffective) breathing, however, is something that you can address. Assist his ventilations with a bag-mask device, keep him warm, and transport without delay. A paramedic intercept should strongly be considered (if available); however, they would NOT administer insulin because that would only lower his blood glucose further. They would administer intravenous glucose.

A 55-year-old woman with a history of diabetes is found unresponsive with rapid, shallow respirations. The patient's husband tells you that he does not know when his wife last took her insulin. Management of this patient should include: A) assisted ventilations and oral glucose. B) assisted ventilations nd rapid transport. C) oral glucose and oxygen via nonrebreathing mask. D) subcutaneous injection of insulin and 100% oxygen.

B) assisted ventilations nd rapid transport. Without knowing if and when the patient last took her insulin, it is difficult to determine if she is experiencing diabetic coma or insulin shock. Nonetheless, her rapid, shallow respirations--which are likely not producing adequate tidal volume--should be treated with ventilation assistance. Because she is unresponsive and obviously unable to swallow, oral glucose is contraindicated. Assess the patient's blood glucose level and pass this information along to the hospital; if possible, arrange for a paramedic intercept so she can receive intravenous dextrose. If the patient is experiencing diabetic coma, insulin is what she truly needs; however, insulin is rarely, if ever, administered in the prehospital setting, even by paramedics. After ensuring adequate oxygenation and ventilation, transport the patient without delay.

Activated charcoal is contraindicated for a patient who is: A) emotionally upset and has ingested two bottles of aspirin. B) awake and alert and has swallowed a commercial drain cleaner. C) agitated and claims to have ingested a bottle of Tylenol. D) conscious and alert and has ingested a large amount of Motrin.

B) awake and alert and has swallowed a commercial drain cleaner. Activated charcoal adsorbs (binds to) many ingested substances, preventing them from being absorbed into the body by the stomach or intestines. In some cases, you may give activated charcoal to patients who have ingested certain substances, if approved by medical control or local protocol. Activated charcoal is contraindicated for patients who have ingested an acid or alkali (ie, drain cleaner) or a petroleum product (ie, gasoline), who have a decreased level of consciousness and cannot protect their own airway, or who are unable to swallow.

A 30-year-old man with a history of schizophrenia cut his wrists and is bleeding profusely. He is confused and combative and has slurred speech. With the assistance of law enforcement personnel, you and your partner physically restrain him so that you can provide care and transport. In this situation, a court of law would MOST likely: A) determine that the patient had decision-making capacity. B) consider your actions in providing care to be appropriate. C) conclude that you should have had a court order to restrain the patient. D) agree that you and your partner are guilty of assault and battery.

B) consider your actions in providing care to be appropriate. An adult with decision-making capacity (ie, a mentally competent adult) has the legal right to refuse medical treatment, even if that treatment involves lifesaving care. In psychiatric cases, however, a court of law would likely consider your actions in providing lifesaving care to be appropriate, particularly if you have a reasonable belief that the patient will harm himself or herself or others without your intervention. In addition, a patient who is in any way impaired, whether by mental illness, medical condition, or intoxication, may not be considered competent to refuse treatment and transport. If you are unsure of a patient's decision-making capacity, err on the side of treatment and transport. Few would argue that it would be easier to defend why you treated a patient than to justify or defend why you abandoned a patient.

The bite of a brown recluse spider is characterized by: A) immediate pain, swelling at the site, and painful muscle spasms. B) delayed onset of pain, swelling, and blister formation at the site. C) two small puncture marks, swelling, and delayed onset of pain. D) rapid swelling within 30 minutes and a decline in mental status.

B) delayed onset of pain, swelling, and blister formation at the site. The venom of a brown recluse spider is cytotoxic; that is, it causes tissue and cellular damage. The bite itself is usually painless; however, the patient typically begins to complain of pain within a few hours. The area becomes swollen and tender, developing a pale, mottled, cyanotic center and possibly a small blister. Over the next several days, a scab of dead skin, fat, and debris will form and dig down into the skin, producing a large ulcer that may not heal unless promptly treated. The bite of a brown recluse spider is very small; therefore, puncture marks are not visible. Unlike the brown recluse spider, the black widow spider bite is characterized by immediate pain. Its venom is neurotoxic; that is, it suppresses the central nervous system. The patient typically complains of intense muscle spasms, especially of the abdomen, and systemic signs of nervous system involvement (eg, dizziness, chest pain, difficulty breathing).

A 36-year-old man who is a known diabetic, presents with severe weakness, diaphoresis, and tachycardia. He is conscious, but confused. His blood pressure is 110/58 mm Hg, pulse is 120 beats/min and weak, and respirations are 24 breaths/min. The glucometer reads error after several attempts to assess his blood glucose level. Medical control will MOST likely order you to: A) assist the patient in taking his insulin. B) give at least one tube of oral glucose C) transport only and closely monitor him. D) give him a salt-containing solution to drink.

B) give at least one tube of oral glucose The patient"s signs and symptoms are consistent with hypoglycemia. When you are in doubt as to a patient's blood glucose level, you should err on the side of caution and give sugar; this is what medical control will likely order you to do. The patient, although confused, is conscious and will likely be able to swallow. Insulin is not administered to patients in the field, even if hyperglycemia is documented; EMTs and paramedics are usually not familiar with all of the different types of insulin and their respective doses, and profound hypoglycemia, potentially resulting in death, can occur if too much insulin is given. After giving oral glucose, reassess the patient's blood glucose level, mental status, and vital signs.

All of the fo llowing are signs of gastrointestinal bleeding, EXCEPT: A) melena. B) hemoptysis. C) tachycardia. D) hematemesis.

B) hemoptysis. Signs and symptoms of gastrointestinal (GI} bleeding include abdominal pain; vomiting blood (hematemesis); the passage of dark, tarry stools (melena); and bright red rectal bleeding (hematochezia). If blood loss is significant, the patient may have signs of shock (eg, tachycardia, diaphoresis, tachypnea, hypotension). Hemoptysis (coughing up blood) is a sign of a pulmonary injury or left side heart failure with pulmonary edema, not GI bleeding.

You are called to a local park for an ill person. It is a hot day and the humidity is high. When you arrive, a bystander directs you to the patient, a young man who is semiconscious. His skin is flushed, hot, and moist. Your FIRST action in the management of this patient should be to: A) ensure an open airway. B) move him to a cool area. C) administer high-flow oxygen. D) begin rapid cooling measures.

B) move him to a cool area. Your first action in a heat-related emergency is to move the patient to a cooler environment. Once you have moved the patient to a cooler place, you should begin your assessment and treat the patient accordingly. Remember, you must FIRST prevent further harm to the patient. The patient in this scenario has signs of heatstroke. After ensuring airway patency, adequate breathing, and adequate circulation, you should proceed with rapid cooling measures.

A 50-year-old woman with a history of epilepsy is actively seizing. Care for this patient should focus primarily on: A) frequently suctioning her airway and carefully restraining her. B) protecting her from injury and ensuring adequate ventilation. C) administering high-flow oxygen and requesting an ALS ambulance. D) placing a bite block in between her molars and giving her oxygen.

B) protecting her from injury and ensuring adequate ventilation. Seizure deaths are most frequently the result of hypoxia. When a person is actively seizing, he or she is not breathing adequately. Your primary focus when treating a seizure patient is to protect him or her from injury and to ensure adequate ventilation and oxygenation. Many seizing patients require assisted ventilation. Suction the oropharynx only if the patient has secretions in his or her mouth. Do NOT insert anything into the mouth of a seizing patient; doing so may cause an airway obstruction or damage the soft tissues of the mouth, resulting in bleeding. Do not attempt to restrain an actively seizing patient; doing so may result in musculoskeletal injuries. Request an ALS ambulance per your local protocols.

A 56-year-old diabetic man is found unresponsive by his wife. She tells you that he ate breakfast this morning, but is unsure if he took his insulin. His respirations are rapid and shallow, his skin is cool and profusely diaphoretic, and his pulse is rapid and weak. Which of the following statements regarding this patient is correct? A) The fact that he ate breakfast makes hypoglycemia highly unlikely. B) He probably did not take his insulin and has a high blood glucose level. C) You should request an ALS unit so they can give the patient his insulin. D) He needs glucose as soon as possible because he is likely hypoglycemic.

D) He needs glucose as soon as possible because he is likely hypoglycemic. Based on the patient's presentation, you should suspect that he is in insulin shock (hypoglycemic crisis) and requires glucose as soon as possible. Cool, clammy (diaphoretic) skin; rapid, shallow respirations; and a rapid, weak pulse are classic signs of insulin shock. Diabetic coma (hyperglycemic ketoacidosis) typically presents with deep, rapid respirations (Kussmaul respirations); warm, dry skin; and tachycardia. If a diabetic patient has an altered mental status or is unresponsive, you should assume that he or she is hypoglycemic unless there is a specific information to the contrary (ie, failure to take prescribed insulin, hypoglycemia ruled out by glucometer). Even if the patient is in diabetic coma and needs insulin, it is rarely given in the field by ALS personnel. The EMT can quickly rule hypoglycemia in or out by assessing the patient's blood glucose level with a glucometer.

Which of the following conditions would place the patient at greatest risk for complications after receiving epinephrine? A) Asthma B) Bradycardia C) Hypovolemia D) Heart disease

D) Heart disease While there are no contraindications for epinephrine in a life-threatening situation such as anaphylaxis, the EMT should be aware that epinephrine could cause complications in patients with heart disease or acute coronary syndrome. Epinephrine increases the heart rate, which increases cardiac oxygen demand and consumption. This could exacerbate underlying cardiac disease. The EMT should therefore consult with medical control when administering epinephrine to such patients. In addition to increasing the heart rate and causing vasoconstriction, epinephrine also dilates the bronchioles; this would be a desirable effect in patients with asthma. Hypovolemia should be treated initially with IV fluids; however, epinephrine (or other similar drugs) may be required if IV fluids alone do not improve perfusion.

After several months of using an illegal substance, the patient develops a tolerance for it. What does this mean? A) The drug achieves its effect at the same dose every time it is used B) An overwhelming need to use the drug causes more frequent dosing C) The drug is used more frequently in order to achieve a greater effect D) Increasing amounts of the drug are needed to achieve the same effect

D) Increasing amounts of the drug are needed to achieve the same effect Over time, a person who routinely misuses a substance may need increasing amounts of it to achieve the same effect. This is called developing a tolerance to the substance. A person with an addiction has an overwhelming desire or need to continue using the substance at whatever the cost, with a tendency to increase the dose. This does not only happen with the classic drugs of abuse, such as heroin or cocaine. Almost any substance can be abused, including laxatives, nasal decongestants, vitamins, and food.

Which of the following complications would MOST likely occur if a patient with end-stage renal disease missed several dialysis treatments? A) Severe dehydration B) Potassium depletion C) Bacterial infection D) Pulmonary edema

D) Pulmonary edema Dialysis removes waste products from the body when the patient's kidneys are unable to do so on their own. If dialysis is not performed, everything that should have been removed remains in the body. This can lead to myriad problems for the patient. Potassium levels can rise to dangerous levels (hyperkalemia), and the patient can develop volume overload; this can result in pulmonary edema. Dehydration would be more likely to occur with aggressive dialysis, not missing a treatment. Patients receiving dialysis are at increased risk for infection, not because of the dialysis, but because of the underlying renal disease.

When you arrive at a residence for a man who is "not acting right," you enter the house and find him sitting on his couch. Which of the following findings would be MOST indicative of an altered mental status? A) Odor of alcohol B) Closed eyes C) Tired appearance D) Slurred speech

D) Slurred speech Often, an altered mental status can be difficult to assess, especially if you do not know how the patient normally acts. However, there are key findings that should increase your index of suspicion. An abnormal speech pattern, such as slurring or incoherent words, can be the result of a diabetic problem, alcohol intoxication, or drug ingestion. All of these can cause an altered mental status. The odor of alcohol suggests intoxication as a potential cause of his problem, but cannot be quantified. Just because the patient's eyes are closed or he has a tired appearance, does not necessarily indicate that he has an altered mental status.

You receive a call to a restaurant where a 34-year-old man is experiencing shortness of breath. When you arrive, you immediately note that the man has urticaria on his face and arms. He is conscious, but restless, and is in obvious respiratory distress. You should: A) ask the patient if he has an epinephrine auto-injector. B) remove the patient's shirt to inspect his chest for urticaria. C) obtain vital signs and a SAMPLE history. D) administer supplemental oxygen at 12 to 15 L/min.

D) administer supplemental oxygen at 12 to 15 L/min. Initial management of a patient with a severe allergic reaction is to ensure a patent airway and administer supplemental oxygen. Positive pressure ventilation may be required if the patient is breathing inadequately. After ensuring a patent airway and adequate ventilation and oxygenation, you should inquirewhether the patient has a prescribed epinephrine auto- injector. If so, you should contact medical control and obtain permission (if required by local protocol) to assist the patient with the auto-injector.

A 44-year-old woman was bitten on the ankle by an unidentified snake while working in her garden. She is conscious and alert, has stable vital signs, and denies shortness of breath. Her only complaint is a burning sensation at the wound site. Your assessment reveals two small puncture wounds, redness, and swelling. You should: A) elevate her leg 12 inches and apply an ice pack to reduce pain and swelling. B) apply a constricting band proximal to the bite and use ice to prevent venom spread. C) conclude that envenomation did not occur and allow a friend to take her to the hospital. D) splint her leg to decrease movement and keep her leg below the level of her heart.

D) splint her leg to decrease movement and keep her leg below the level of her heart. Given the fact that the snake was not identified, you should assume that it was poisonous. Furthermore, the presence of puncture wounds, burning, redness, and swelling are suggestive of envenomation. Therefore, you should provide emergency care and transport the patient to the hospital. Treatment for a snake bite involves keeping the patient calm, administering oxygen (if hypoxemic or short of breath), splinting the affected extremity to decrease movement (helps slow the spread of venom), keeping the extremity below the level of the heart, and transporting the patient to the hospital. Do NOT apply ice to a snake bite; it may constrict the blood vessels and force venom further into the bloodstream. The use of a proximal constricting band is controversial; if one is used, it should be loose, not tight. En route to the hospital, monitor the patient's vital signs and mental status, and be alert for vomiting.

When assessing a patient with an apparent behavioral crisis, the MOST important initial observation the EMT should make is whether or not: A) the patient is oriented to person, place, and time. B) the patient's residence is well kept or in disarray. C) there is any drug paraphernalia near the patient. D) the patient seems aggressive or verbally abusive.

D) the patient seems aggressive or verbally abusive. All of the options in this question are important to assess. However, the safety of you and your crew supersedes everything! If ANY patient is aggressive or verbally abusive, that is a direct threat to your safety, and you should request law enforcement personnel. It is common (and wise) for law enforcement to be dispatched to the scene automatically for patients with behavioral crises. If they are not, however, you should request them as soon as possible. Once it is safe for you and your team to do your job, you should gather information specific to the patient's chief complaint or presentation.


Related study sets

Anthropology 1050 Exam 3 Study Guide Chapter 10

View Set

Microeconomics Chapter 1 Midterm

View Set

ACCT 315 Test 4, Unit 7: Agency, Test 4, BLaw, BLAW Test 4 Practice

View Set

Human Resource Final Exam True and False

View Set

DECA Business Finance Series - Financial Analysis Performance Indicators (Tiers 1, 2, and 3)

View Set

Athletic Injuries Chapter 11 Review Questions

View Set

LSAT: Premise & Conclusion Indicators

View Set

Chapter 1 Introduction PMP 6th edition

View Set

Chapter 7: The Foundation of Savings

View Set